Feb 2016 MEE

Download as pdf or txt
Download as pdf or txt
You are on page 1of 48

February 2016

MEE Questions
and Analyses

National Conference of Bar Examiners


302 South Bedford Street | Madison, WI 53703-3622
® Phone: 608-280-8550 | Fax: 608-280-8552 | TDD: 608-661-1275
www.ncbex.org e-mail: [email protected]
Copyright © 2016 by the National Conference of Bar Examiners.
All rights reserved.
Contents

Preface.............................................................................................................................................ii

Description of the MEE ..................................................................................................................ii

Instructions.....................................................................................................................................iii

February 2016 Questions

Secured Transactions Question.................................................................................................3

Evidence Question ....................................................................................................................4

Agency & Partnership Question ...............................................................................................6

Constitutional Law Question ....................................................................................................7

Decedents’ Estates Question.....................................................................................................8

Family Law Question................................................................................................................9

February 2016 Analyses

Secured Transactions Analysis ...............................................................................................13

Evidence Analysis...................................................................................................................16

Agency & Partnership Analysis..............................................................................................21

Constitutional Law Analysis...................................................................................................25

Decedents’ Estates Analysis ...................................................................................................29

Family Law Analysis ..............................................................................................................33

i
Preface

The Multistate Essay Examination (MEE) is developed by the National Conference of Bar
Examiners (NCBE). This publication includes the questions and analyses from the February
2016 MEE. (In the actual test, the questions are simply numbered rather than being identified by
area of law.) The instructions for the test appear on page iii.

The model analyses for the MEE are illustrative of the discussions that might appear in excellent
answers to the questions. They are provided to the user jurisdictions to assist graders in grading
the examination. They address all the legal and factual issues the drafters intended to raise in the
questions.

The subjects covered by each question are listed on the first page of its accompanying analy­
sis, identified by roman numerals that refer to the MEE subject matter outline for that subject.
For example, the Constitutional Law question on the February 2016 MEE tested the following
areas from the Constitutional Law outline: II.A.1. The separation of powers—The powers of
Congress—Commerce, taxing, and spending powers.

For more information about the MEE, including subject matter outlines, visit the NCBE website
at www.ncbex.org.

Description of the MEE

The MEE consists of six 30-minute questions and is a component of the Uniform Bar
Examination (UBE). It is administered by user jurisdictions as part of the bar examination
on the Tuesday before the last Wednesday in February and July of each year. Areas of law
that may be covered on the MEE include the following: Business Associations (Agency and
Partnership; Corporations and Limited Liability Companies), Civil Procedure, Conflict of Laws,
Constitutional Law, Contracts, Criminal Law and Procedure, Evidence, Family Law, Real
Property, Secured Transactions (UCC Article 9), Torts, and Trusts and Estates (Decedents’
Estates; Trusts and Future Interests). Some questions may include issues in more than one area of
law. The particular areas covered vary from exam to exam.

The purpose of the MEE is to test the examinee’s ability to (1) identify legal issues raised by a
hypothetical factual situation; (2) separate material which is relevant from that which is not; (3)
present a reasoned analysis of the relevant issues in a clear, concise, and well-organized compo­
sition; and (4) demonstrate an understanding of the fundamental legal principles relevant to the
probable solution of the issues raised by the factual situation. The primary distinction between
the MEE and the Multistate Bar Examination (MBE) is that the MEE requires the examinee to
demonstrate an ability to communicate effectively in writing.

ii
Instructions

The back cover of each test booklet contains the following instructions:

You will be instructed when to begin and when to stop this test. Do not break the seal on
this booklet until you are told to begin.

You may answer the questions in any order you wish. Do not answer more than one ques­
tion in each answer booklet. If you make a mistake or wish to revise your answer, simply
draw a line through the material you wish to delete.

If you are using a laptop computer to answer the questions, your jurisdiction will provide
you with specific instructions.

Read each fact situation very carefully and do not assume facts that are not given in the
question. Do not assume that each question covers only a single area of the law; some of
the questions may cover more than one of the areas you are responsible for knowing.

Demonstrate your ability to reason and analyze. Each of your answers should show an
understanding of the facts, a recognition of the issues included, a knowledge of the appli­
cable principles of law, and the reasoning by which you arrive at your conclusions. The
value of your answer depends not as much upon your conclusions as upon the presence
and quality of the elements mentioned above.

Clarity and conciseness are important, but make your answer complete. Do not volunteer
irrelevant or immaterial information.

Answer all questions according to generally accepted fundamental legal principles


unless your testing jurisdiction has instructed you to answer according to local case or
statutory law.

NOTE: Examinees testing in UBE jurisdictions must answer according to generally


accepted fundamental legal principles rather than local case or statutory law.

iii
February 2016
MEE Questions
Secured Transactions

Evidence

Agency & Partnership

Constitutional Law

Decedents’ Estates

Family Law
SECURED TRANSACTIONS QUESTION
Two years ago, a retailer of home electronic equipment borrowed $5 million from a finance com­
pany. The loan agreement, signed by both parties, provided that the retailer granted the finance
company a security interest in all of the retailer’s present and future inventory to secure the
retailer’s obligation to repay the loan. On the same day that it made the loan, the finance com­
pany filed in the appropriate state filing office a properly completed financing statement reflect­
ing this transaction.

Six months ago, a buyer purchased a home entertainment system from the retailer for a total
price of $7,000. The buyer paid $1,000 as a down payment on the system and agreed to make
12 additional monthly payments of $500 each. The buyer signed a “credit purchase agreement”
memorializing the financial arrangement with the retailer and providing that the retailer would
“retain title” to the entertainment system until the buyer’s obligation to the retailer was paid in
full. The buyer then returned home with her new home entertainment system. The buyer had
no knowledge of the retailer’s agreement with the finance company and acted in good faith in
acquiring the home entertainment system. The retailer did not file a financing statement with
respect to this transaction.

Two months ago, the buyer decided that she could no longer afford her monthly $500 payments
for the home entertainment system. She contacted her friend, who had often expressed interest
in acquiring a home entertainment system. After a brief discussion, the friend agreed to buy the
home entertainment system from the buyer for $4,000 if the friend could pay the price 90 days
later, when he anticipated receiving a bonus at work. The buyer accepted the friend’s proposal,
and the friend gave the buyer a check for $4,000. The buyer promised to hold the $4,000 check
for 90 days before depositing it. The friend took the entertainment system and began using it at
his own home. The friend had no knowledge of the buyer’s agreement with the retailer or of the
retailer’s agreement with the finance company.

The retailer is in financial distress and has missed a payment owed to the finance company.
Meanwhile, since the friend bought the home entertainment system from the buyer, the buyer has
not made any of her monthly payments to the retailer.

1. Does the finance company have an interest in the home entertainment system? Explain.

2. Does the retailer have an interest in the home entertainment system? Explain.

3. Does the retailer have an interest in the $4,000 check? Explain.

3
EVIDENCE QUESTION
A victim had just walked out of a jewelry store carrying a package containing a diamond bracelet
when someone grabbed him from behind, put a gun to his back, and demanded the package. The
victim handed the package over his shoulder to the robber. The robber said, “Close your eyes and
count to 20. I’ll be watching, and if you mess up, I’ll shoot you.” The victim did as he was told,
and when he opened his eyes, the robber was gone. The victim immediately called 911 on his
cell phone.

The victim did not see the robber. A witness on the other side of the street saw the entire
encounter. While the victim was speaking to the 911 operator, the witness ran over to the victim
and shouted, “Are you all right? I saw it all!”

A police officer arrived five minutes later and took a statement from the witness, who was
wringing her hands and pacing. The police officer asked the witness, “What did you see?” The
witness responded, “The robber is about six feet tall. He has brownish hair, almost buzzed to the
scalp. He was wearing jeans and a blue jacket.” The police officer called in the description to the
police station.

The defendant, who is over six feet tall and has buzzed brown hair, was picked up 30 minutes
later. When the police officer stopped him, he was six blocks from the scene of the robbery.
The defendant was wearing jeans and a blue jacket but did not have a gun or the bracelet in his
possession. He was brought to the police station for questioning and was placed in a lineup.

The police officer brought the witness to the police station to view the lineup. The witness
viewed the lineup and identified the defendant as the robber. The defendant was arrested and
charged with robbery.

One week after the robbery, the witness moved overseas. One year later, at the time of the defen­
dant’s trial, the witness could not be found.

The victim and the police officer both testified at trial for the prosecution. The police officer
testified as follows:

Question: When you arrived at the scene of the robbery, did you obtain a description of the
robber?

Answer: Yes. The witness said that the robber was about six feet tall, with very short,
brownish hair, almost buzzed to the scalp, and that he was wearing jeans and a blue jacket.

Question: Did you gather any other evidence indicating that the defendant committed this
robbery?

Answer: Yes. When I was walking into the police station with the victim, we overheard
the defendant in an adjoining room. As soon as the victim heard the defendant’s voice, the
victim said, “That’s the voice of the guy who robbed me.”

4
Evidence Question

Question: What do you know about the defendant?

Answer: He’s a known drug dealer who had been hanging around in the area where the
jewelry store is located for six months before the robbery, constantly causing trouble.

The trial was held in a jurisdiction that has rules identical to the Federal Rules of Evidence.
Defense counsel made timely objections to the admission of the following evidence:
(a) The police officer’s testimony recounting the witness’s statement at the scene.
(b) The police officer’s testimony recounting the victim’s statement while walking into the
police station.
(c) The police officer’s testimony that the defendant is a “known drug dealer who had
been hanging around in the area where the jewelry store is located for six months
before the robbery, constantly causing trouble.”

The trial judge overruled all of defense counsel’s objections.

Was this evidence properly admitted? Explain.

5
AGENCY & PARTNERSHIP QUESTION
Four years ago, a man and a woman properly formed a partnership to own and manage a multi­
million-dollar apartment complex. They qualified the partnership as a limited liability partner­
ship (LLP). The complex required a good deal of maintenance, and they anticipated regular
borrowings of up to $25,000 to cover maintenance expenses as is customary in this industry.

While the partnership agreement contained no limitations on the authority of the partners to act
for LLP, two months after LLP was formed the man and the woman agreed that neither partner
would have authority to incur indebtedness on behalf of LLP in excess of $10,000 without the
consent of the other partner. They then signed a statement of partnership authority describing this
limitation, but this statement was never filed.

Over the next two years, the man regularly borrowed amounts from LLP’s bank to cover the
complex’s ordinary maintenance expenses. The amounts borrowed ranged from $5,000 to
$9,000, and the man did not ask for the woman’s consent when he entered into these loans on
behalf of LLP.

Earlier this year, the man, without the woman’s knowledge, asked the bank to loan $25,000 to
LLP. The man told the bank’s loan officer that the funds would be used for ordinary maintenance
of the apartment complex. This amount, though greater than LLP’s previous borrowings from the
bank for maintenance, was in line with loans made by the bank for maintenance to other similar
apartment complexes.

When the loan officer asked the man if he had authority to borrow the money on behalf of LLP,
the man handed the loan officer a copy of the partnership agreement. The man, however, did not
give the officer a copy of the statement of partnership authority, nor did he tell the loan officer
that it existed. The bank had no actual knowledge of the limitation on the man’s authority to
obtain the loan on behalf of LLP.

Without contacting the woman, the bank loaned $25,000 to LLP. The loan agreement was signed
only by the man and the bank’s loan officer. The woman, though she had knowledge of the ear­
lier borrowings from the bank, had no knowledge of this loan.

The man then used the $25,000 to pay his personal gambling debts. LLP has not made any pay­
ments to the bank on the loan.

1. Is LLP liable to the bank on the loan? Explain.

2. Is the woman personally liable to the bank on the loan? Explain.

3. Is the man liable for breaching his fiduciary duties and, if so, to whom is he liable?
Explain.

6
CONSTITUTIONAL LAW QUESTION
State A, a leader in wind energy, recently enacted the “Green Energy Act” (“the Act”).

Section 1 of the Act requires that 50% of the electricity sold by utilities in the state come from
“environmentally friendly energy sources.” Wind energy, which is produced in State A, is
classified by the Act as an “environmentally friendly energy source.” Natural gas, which is not
produced in State A, is not classified by the Act as environmentally friendly. The preamble of
the Act contains express findings that the burning of natural gas releases significant quantities
of greenhouse gases into the atmosphere and requires the diversion of scarce water resources for
use in gas-burning thermoelectric plants.

Section 2 of the Act prohibits the Public Service Commission of State A from approving any
new coal-burning power plants in the state, unless it finds that “the construction of the plant is
necessary to meet urgent energy needs of this state.” A public utility in neighboring State B has
applied for a permit to build a coal-burning power plant on property it owns across the border
in State A. The Commission has denied the utility’s application based on its finding that there
is no evidence of any urgent energy needs in State A. The State B utility presented undisputed
evidence of severe energy shortages in State B, but the Commission rejected this evidence as
irrelevant to the statutory exception.

Section 3 of the Act requires State A, whenever possible, to buy goods and services only from
“environmentally friendly vendors located within the state.” To qualify as an “environmentally
friendly vendor,” a firm must meet specified standards concerning energy efficiency, chemical
use, and use of recycled materials. A vendor located outside of State A meets all the standards to
qualify as an environmentally friendly vendor. The vendor has sought to sell goods and services
to State A. The relevant State A agencies have refused to purchase from this vendor, pointing out
that the Act requires them to purchase, if possible, only from “environmentally friendly vendors
located within the state,” of which there are several.

There is no federal statute or regulation relevant to this problem.

Which provisions, if any, of the Green Energy Act unconstitutionally burden or discriminate
against interstate commerce? Explain.

7
DECEDENTS’ ESTATES QUESTION
Last year, a patient, age 80, was diagnosed with cancer. Shortly after receiving the cancer
diagnosis, the patient signed a durable health-care power of attorney (POA) designating her son
as her “agent to make all health-care decisions on my behalf when I lack capacity to make them
myself.” The POA contained no other provisions relevant to the commencement or duration of
the agent’s authority. The patient thereafter underwent several cancer therapies which were so
successful that, two months ago, the patient’s doctor said that, in his opinion, the patient’s cancer
was in “complete remission.”

Last week, the patient was struck by an automobile, suffered serious injuries to her head and
neck, and underwent emergency surgery for those injuries. Following surgery, the patient’s
doctor explained to her son that there was a more than 50% risk that the patient would not regain
consciousness and would need to be maintained on life-support systems to provide her with food,
hydration, and respiration. The doctor also noted that, during the next few days, there was a large
risk of a stroke or cardiac arrest, which would substantially increase the risk that the patient
would never regain consciousness, and which could be fatal.

The patient’s son was confident that his mother would not want to be kept on life support if
she were permanently unconscious but believed that she would want to be maintained on life
support until her status was clear. He thus instructed the doctor to put the patient on life support
but not to resuscitate her if she were to experience a stroke or cardiac arrest. The son issued
these instructions after conferring with the doctor and with his two sisters. The sisters disagreed
with their brother’s decision and told the doctor to ignore the instructions “because we have as
much right to say what happens to Mom as he does, and we want her resuscitated in all events.”
Nonetheless, the doctor thereafter placed a “do not resuscitate” (DNR) order in the patient’s
chart.

Four days ago, the patient, who had not regained consciousness, suffered a cardiac arrest.
Following the DNR order, the nursing staff did not attempt to resuscitate the patient, and she
died.

The patient’s valid will devised her estate to her three children in equal shares. All three children
survived the patient.

This jurisdiction has a typical statute authorizing durable health-care powers of attorney. This
jurisdiction also has a statute providing that “[n]o person shall share in the estate of a decedent
when he or she intentionally caused the decedent’s death.”

The patient’s two daughters have consulted an attorney, who has advised them that (1) the
patient’s son had no authority to instruct the doctor to write the DNR order; (2) in a wrongful
death action, the son would be liable for the patient’s death; and (3) the son is barred from taking
under the patient’s will because his actions intentionally caused her death.

Is the attorney correct? Explain.

8
FAMILY LAW QUESTION
Eight years ago, a woman and a man began living together. The woman worked as an investment
banker, and the man worked part-time as a bartender while he struggled to write his first novel.
The couple lived in a condominium that the woman had purchased shortly before the man moved
in. The woman had purchased the condominium for $300,000 using her own money and had
taken title in her own name.

Four years ago, the woman and the man were married at City Hall. One week before the
wedding, the woman presented the man with a proposed premarital agreement and an asset list.
The asset list correctly stated that the woman owned the condominium, then worth $350,000, and
a brokerage account, then worth $500,000. The agreement specified that, in the event of divorce,
each spouse would be entitled to retain “all assets which he or she then owns, whether or not
those assets are acquired during the marriage.” The man was surprised when the woman gave
him the agreement to sign, and he contacted a lawyer friend for advice. The lawyer urged the
man not to sign the agreement. Nonetheless, the man signed the agreement, telling the woman,
“I’m a little hurt, but I guess I understand that you want to keep what you earn.” The woman
signed the agreement as well.

After their wedding, the woman and the man continued to live in the woman’s condominium and
to work at the jobs each held before the marriage. The man also continued to work on his novel.

Six months ago, the man’s novel was accepted by a publisher. The novel will be released next
spring. The publisher has estimated that the royalties may total as much as $200,000 over the
next five years.

Two months ago, the woman and the man separated. The woman remained in the condominium,
now worth $400,000 as a result of market appreciation. The woman’s brokerage account,
worth $500,000 when she and the man married, is now worth $1,000,000 as a result of market
appreciation and additional investments that the woman made with employment bonuses she
received during the marriage. The woman has made no withdrawals from this account.

One month ago, the woman won, but has not yet received, a $5 million lottery jackpot.

One week ago, the man filed for divorce. In the man’s divorce petition, he asks the court to
invalidate the premarital agreement and seeks half of all assets owned by the woman, i.e., the
woman’s brokerage account, her condominium, and her right to the lottery payment. The man
owns no assets except for personal effects and the book contract under which he will receive
future royalties based on sales of his novel.

This jurisdiction has adopted the Uniform Premarital Agreement Act, which in relevant part
provides that “the party against whom enforcement [of the premarital agreement] is sought
must prove (1) involuntariness or (2) both that ‘the agreement was unconscionable when it was
executed’ and that he or she did not receive or waive a ‘fair and reasonable’ disclosure and ‘did
not have or reasonably could not have had . . . an adequate knowledge’ of the other’s assets and
obligations.”

9
Family Law Question

The jurisdiction’s divorce law requires “equitable distribution” of all marital (community) assets
and prohibits the division of separate assets.

1. Is the premarital agreement enforceable? Explain.

2. Assuming that the agreement is unenforceable, what assets are subject to division in the
divorce action, and what factors should a court consider in distributing those assets? Explain.

10
February 2016
MEE Analyses
Secured Transactions

Evidence

Agency & Partnership

Constitutional Law

Decedents’ Estates

Family Law
SECURED TRANSACTIONS ANALYSIS
Secured Transactions I.B.; II.D., E.; III.B.; IV.A., B., C.

ANALYSIS
Legal Problems:
(1) Does a security interest in a store’s inventory survive sale of an item of that inventory
to a buyer who acquires the item in good faith in an ordinary-course transaction?
(2) What are the rights of a seller of goods who “sells” those goods on credit to a consumer
but purports to retain title to the goods until the full purchase price is paid? Do the
seller’s rights survive a resale of those goods by the buyer to another consumer?
(3) Does a secured party obtain an interest in the proceeds of a debtor’s sale of the secured
party’s collateral?

DISCUSSION
Summary

The finance company had a perfected security interest in the home entertainment system when it
was owned by the retailer. The buyer acquired the home entertainment system from the retailer
free of that security interest because the buyer was a buyer in ordinary course of business.
Because the buyer owned the home entertainment system free of the security interest, the buyer
sold it to her friend free of the finance company’s interest.

The retailer had a perfected security interest in the home entertainment system while it was
owned by the buyer. The friend acquired the home entertainment system from the buyer free of
that security interest because both the buyer and the friend used the home entertainment system
for personal purposes, the friend acquired it without knowledge of the retailer’s security interest,
and the retailer had not filed a financing statement with respect to its security interest.

Because the retailer had a security interest in the home entertainment system when it was owned
by the buyer, the retailer obtained a security interest in the $4,000 check as proceeds of the home
entertainment system. The retailer’s security interest in the check was perfected because its secu­
rity interest in the home entertainment system had been perfected and the check constituted “cash
proceeds.”

Point One (40%)

The buyer obtained the home entertainment system free of the finance company’s perfected
security interest because the buyer was a buyer in ordinary course of business. Because the buyer
owned the home entertainment system free of the security interest, the buyer’s sale to the friend
was free of that interest.

The finance company had an enforceable, attached security interest in all of the retailer’s inven­
tory, including the home entertainment system later sold to the buyer. All three elements of
enforceability and attachment under UCC § 9-203 were satisfied: value was given (the loan),
13
Secured Transactions Analysis

the debtor (the retailer) had rights in the inventory, and the debtor (the retailer) authenticated a
security agreement containing a description of the collateral. See UCC §§ 9-203(b), 9-203(a),
and 9-108. Moreover, the finance company’s security interest was perfected by the filing of the
financing statement. See UCC §§ 9-308, 9-310.

As a general rule, a security interest continues notwithstanding the sale of collateral. UCC
§ 9-315(a)(1). But this general rule is subject to several exceptions. In particular, a “buyer in
ordinary course of business” (BIOCOB) takes free of a security interest created by its seller
even if the security interest was perfected. UCC § 9-320(a). Under these facts, the buyer was a
BIOCOB. She appears to have bought the home entertainment system in good faith and in the
ordinary course from a person in the business of selling goods of the kind (the retailer) without
knowledge that the sale to her violated the rights of a third party. UCC § 1-201(b)(9). Because
the buyer was a BIOCOB, she acquired the home entertainment system free of the finance com­
pany’s security interest.

Under the “shelter principle,” once the buyer acquired the home entertainment system free of the
finance company’s security interest, any subsequent transfer of the system by the buyer to some­
one else was also free of the finance company’s security interest. Accordingly, when the friend
acquired the home entertainment system from the buyer, it was free of the finance company’s
security interest. See Frederick Miller, carl Bjerre, and neil cohen, 9B hawkland Ucc
SerieS § 9-320:2 [Rev.]. See also PEB Commentary No. 6, Section 9- 301(1) (March 10, 1990);
UCC § 2-403(1).

Point Two (40%)

The retailer’s retention of title to the home entertainment system in the credit sale to the buyer
was a security interest in the system. While the security interest was automatically perfected, the
friend acquired the system free of the retailer’s security interest.

While the agreement between the retailer and the buyer indicated that the retailer was retaining
title to the home entertainment system, the substance-over-form rules of secured transactions law
indicate that the retailer’s interest was not ownership but, rather, a security interest. See UCC
§ 1-201(b)(35) [1-201(37) in Missouri, which has not yet enacted revised Article 1 of the UCC];
see also UCC § 9-109(a)(1) (Article 9 governs any transaction regardless of form that creates a
security interest in personal property by contract). All the elements of attachment and enforce­
ability of this security interest were satisfied. (See UCC § 9-203 and the analysis in Point One
above.)

The retailer’s security interest was perfected even though the retailer did not file a financing
statement. This is because the retailer’s security interest was a “purchase-money security inter­
est” in “consumer goods.” The security interest was a purchase-money security interest because
it was retained by the seller to secure payment of the remainder of the sales price. See UCC
§ 9-103. The home entertainment system constituted “consumer goods” because it was used pri­
marily for personal, family, or household purposes. See UCC § 9-102(a)(23). A purchase-money

14
Secured Transactions Analysis

security interest in consumer goods perfects upon attachment without the filing of a financing
statement. UCC § 9-309(1). Therefore, the retailer’s security interest in the home entertainment
system was a perfected security interest while the system was owned by the buyer.

As a general rule, a security interest continues notwithstanding the sale of collateral. UCC
§ 9-315(a)(1). Thus, under this general rule, the retailer’s security interest in the home entertain­
ment system would continue after the buyer’s sale of the system to the friend. But this general
rule is subject to several exceptions. In this case, the friend acquired the system free of the retail­
er’s security interest under the consumer-to-consumer exception in UCC § 9-320(b) because the
home entertainment system was used for personal, family, or household purposes of the buyer
before the sale and of the friend after the sale, and the friend acquired the system for value, with­
out knowledge of the retailer’s security interest, and before a financing statement was filed with
respect to that security interest. (As noted above, the retailer was not required to file a financing
statement to perfect its security interest because it was a purchase-money security interest in con­
sumer goods.)

Point Three (20%)

Because the retailer had a perfected security interest in the home entertainment system at the
time of its sale to the friend, the retailer obtained a security interest in the $4,000 check as pro­
ceeds from the sale of the system; the security interest in the check was perfected for 20 days and
remained perfected thereafter because the check constituted “cash proceeds.”

The check constituted “proceeds” of the home entertainment system because that term includes,
inter alia, whatever is obtained on the sale of collateral. UCC § 9-102(a)(64)(A). When collateral
is disposed of, a secured party automatically obtains a security interest in identifiable proceeds
of the collateral. UCC § 9-315(a)(2). Accordingly, when the home entertainment system was
sold by the buyer to the friend, and the buyer received the check in return, the check constituted
proceeds of the home entertainment system and the retailer (which had a security interest in the
system) thereby obtained a security interest in the check. Because the retailer’s security interest
in the system had been perfected, its security interest in the check as proceeds was perfected as
well. UCC § 9-315(c). That perfection ceases after 20 days, however, unless one of three criteria
in UCC § 9-315(d) is satisfied. In this case, UCC § 9-315(d)(2) was satisfied because the check
was “cash proceeds.” See UCC § 9-102(a)(9). Thus, the retailer’s security interest in the check
was continuously perfected after the sale of the system to the friend.

15
EVIDENCE ANALYSIS
Evidence II.C.1.; V.A.1., B.

ANALYSIS
Legal Problems:
(1)(a) Is a witness’s out-of-court description of a robber provided to the police just minutes
after viewing the robbery inadmissible hearsay?
(1)(b) Does admission of a nontestifying witness’s out-of-court statement, provided in
response to police questioning just minutes after viewing the crime, violate the
Confrontation Clause of the Sixth Amendment?
(2) Is an out-of-court vocal identification of a defendant inadmissible hearsay?
(3) Can the prosecutor introduce testimony that a criminal defendant was a known drug
dealer who had been hanging around for six months constantly causing trouble?

DISCUSSION
Summary

The witness’s out-of-court description of the robber is hearsay. However, the statement fits either
the hearsay exception for present sense impressions or the exception for excited utterances.

Even if the statement falls within a hearsay exception, its admission may have violated the
Confrontation Clause of the Sixth Amendment to the U.S. Constitution. The witness was
unavailable to testify at trial, and the defendant had no opportunity to cross-examine the witness.
Whether admission of this out-of-court statement violated the Confrontation Clause depends on
whether the statement was testimonial. If the objective primary purpose of the witness’s inter­
rogation was to help the police to resolve an ongoing emergency—the need to apprehend an
armed and dangerous robber—the witness’s statement was nontestimonial and the admission
of the statement did not violate the Confrontation Clause. However, if the objective primary
purpose of the witness’s interrogation was to help the police investigation or if the witness rea­
sonably believed that her statement would be used at trial, her statement was testimonial and its
admission violated the Confrontation Clause. A reasonable argument on this point could be made
either way.

Because the victim testified at trial and could have been cross-examined, the officer’s testimony
that the victim had stated “That’s the voice of the guy who robbed me” was admissible as a state­
ment of vocal identification made after perceiving (hearing) the defendant.

The officer’s testimony that the defendant is a “known drug dealer who had been hanging around
in the area where the jewelry store is located for six months before the robbery, constantly caus­
ing trouble” was inadmissible character evidence. This evidence could not be admitted to prove
that the defendant acted in conformity with his character as a drug dealer or criminal. The evi­
dence also could not be admitted to prove other crimes, wrongs, or acts because it was too vague
and served no relevant non-propensity purpose.

16
Evidence Analysis

Point One(a) (35%)

The witness’s description of the robber is hearsay. The police officer’s testimony repeating that
out-of-court statement was properly admitted because the statement was a present sense impres­
sion or an excited utterance.

The witness’s description of the robber is an out-of-court statement. This statement was offered
at trial to prove the truth of the matter asserted in the statement—that the person who perpetrated
the crime matched the witness’s description. Thus, the witness’s statement is inadmissible hear­
say, unless it falls within a hearsay exception. See Fed. r. evid. 801(c).

The witness’s out-of-court statement falls within two hearsay exceptions. Fed. r. evid. 803(1)
and 803(2). Both exceptions apply regardless of whether the witness (the declarant) is available
to testify at trial.

First, the witness’s description is admissible as a “present sense impression.” Rule 803(1) per­
mits the use of a hearsay statement “describing or explaining an event or condition made while
the declarant was perceiving the event or condition or immediately thereafter.” Here, the witness
saw the robbery, ran across the street to the victim, and provided a description of the robber to
the police officer a few minutes later. “A few minutes’ pause . . . is within the period contem­
plated in Rule 803(1).” United States v. Obayagbona, 627 F. Supp. 329, 340 (E.D.N.Y.1985).

Second, the witness’s description of the robber is admissible as an “excited utterance.” See Fed.
r. evid. 803(2). The witness’s description is a statement “relating to a startling event or condi­
tion made while the declarant was under the stress of excitement that it caused.” Id. Here, the
witness had just seen an armed robbery, which is certainly a startling event for victims and
spectators. See, e.g., Cole v. Tansy, 926 F.2d 955 (10th Cir. 1991). The witness’s description of
the robber related to the startling event of the robbery. Moreover, the fact that the witness was
wringing her hands and pacing up and down while she made the statement indicates that she was
still “under the stress of excitement” of the event. See id. at 958. Finally, although the witness’s
description was made in response to the police officer’s question, this normally has no impact on
the operation of the excited utterance exception unless the police questioning was suggestive. See
United States v. Phelps, 168 F.3d 1048, 1055 (8th Cir. 1999).

[NOTE: Rule 801(d)(1)(c) does not apply here because under this rule the declarant must testify
and be subject to cross-examination. See Point Two.]

Point One(b) (20%)

Whether admission of the police officer’s testimony repeating the witness’s description of the
robber violated the Confrontation Clause depends on whether the objective primary purpose
of the interrogation was to assist the police to resolve an ongoing emergency or to assist in
the investigation and prosecution of the crime. Here, the facts support reasonable arguments
either way.

17
Evidence Analysis

The Sixth Amendment of the Constitution gives defendants the right to confront witnesses
against them. The use of an out-of-court statement by the prosecutor violates a defendant’s
Sixth Amendment rights, even if the statement falls within a hearsay exception, if (1) the state­
ment was “testimonial,” (2) the witness who made the statement is unavailable to testify at
trial, and (3) the defendant has not had an opportunity to cross-examine the witness before trial.
See Crawford v. Washington, 541 U.S. 36 (2004); jack B. weinStein & Margaret a. Berger,
5 weinStein’S Federal evidence 804.06[5][a] (2d ed. 1997).

In Crawford, the Supreme Court noted that statements made to police officers in the course of an
interrogation are often testimonial. See id. at 52–53 (“interrogations by law enforcement officers
fall squarely into that class [of testimonial statements]”). The Crawford Court also suggested that
statements that a witness reasonably believed would be used as part of a criminal prosecution are
testimonial. See id. at 51–52 (testimonial statements are “made under circumstances that would
lead an objective witness reasonably to believe that the statement would be available for use at a
later trial”).

Two years later, in Davis v. Washington, 547 U.S. 813 (2006), the Court drew a distinction
between (1) statements made to the police to assist in the investigation and prosecution, which
should be considered testimonial; and (2) statements made to the police to enable them to meet
an ongoing emergency, which should be considered nontestimonial. Thus, after Davis, state­
ments to the police “under circumstances objectively indicating that the primary purpose of the
interrogation is to enable police assistance to meet an ongoing emergency” are nontestimonial.
See id. at 822.

In Michigan v. Bryant, 562 U.S. 344 (2011), the Court elaborated on its objective “primary pur­
pose of the interrogation” standard, noting that it requires “a combined inquiry that accounts for
both the declarant and the interrogator.” Id. at 367. The Bryant Court also set forth a range of
factors that might be used to determine the existence of an “ongoing emergency.” These include
(1) the nature of the dispute,
(2) the scope of the potential harm to the victim,
(3) the threat to additional identifiable victims,
(4) the existence of a more generalized threat to the public,
(5) the suspect’s choice of weapon, and
(6) whether the suspect remained “at large” or had been located (but not yet apprehended) by
the police and/or any other “first responders.”

See id. at 362–78. Finally, in Ohio v. Clark, 135 S.Ct. 2173 (2015), the Court reiterated that “the
question is whether, in light of all the circumstances, viewed objectively, the ‘primary purpose’
of the conversation was to ‘creat[e] an out-of-court substitute for trial testimony.’” Id. at 2180
(quoting Michigan v. Bryant, 562 U.S. at 392).

18
Evidence Analysis

Here, the witness’s out-of-court statement to the police officer could be viewed as nontestimo­
nial because it was made under emergency circumstances. A robbery had recently occurred, the
dispute was not private, the suspect was apparently armed, he may have posed a threat to other
victims and/or the general public, and he was still “at large.” Under these circumstances, the
“primary purpose of the interrogation,” from the perspective of the reasonable declarant and the
interrogator, could have been to assist the police to resolve an ongoing emergency. If the state­
ment is deemed nontestimonial, its admission would not violate the Confrontation Clause.

On the other hand, there are facts that suggest that the witness’s statement was testimonial. The
witness provided the robber’s identity in response to police questioning about a crime, the robber
was no longer present at the scene, and there was no clear ongoing emergency insofar as the wit­
ness was concerned. Based on these facts, the judge could conclude that the objective “primary
purpose of the interrogation” was to assist in the investigation and prosecution of the robbery.
Because the witness was not available to testify and the defendant had no prior opportunity to
cross-examine the witness, if the statement were deemed testimonial, its admission would violate
the Confrontation Clause.

The facts support reasonable arguments either way on this issue.

Point Two (20%)

The victim’s statement “That’s the voice of the guy who robbed me” is admissible as an out-of-
court statement of identification.

The victim’s statement, “That’s the voice of the guy who robbed me,” is an out-of-court state­
ment offered for the truth of the matter asserted. Nonetheless, “[a] statement . . . is not hearsay
[when] [t]he declarant testifies and is subject to cross-examination about a prior statement, and
the statement . . . identifies a person as someone the declarant perceived earlier.” Fed. r. evid.
801(d)(1)(C). Most courts have found that statements identifying a person after hearing the per­
son’s voice qualify as statements of identification under Rule 801(d)(1)(C). See United States v.
Ramirez, 45 F.3d 1096, 1101 n.1 (7th Cir. 1995). Lay witness voice identification based on prior
familiarity with a voice is typically admissible unless it has been tainted by impermissibly sug­
gestive questioning or investigative procedures. See United States v. Garcia-Alvarez, 541 F.3d 8
(1st Cir. 2008). Expert testimony is not normally required.

Here, because the victim testified at trial and could have been recalled and cross-examined con­
cerning his out-of-court statement, the statement was correctly admitted under Rule 801(d)(1)(C)
as a statement of identification.

The victim’s statement is not hearsay under the rules, so it need not fit any of the hearsay excep­
tions to be admissible. However, credit could be given to examinees who conclude that the
victim’s statement falls within the hearsay exception for an excited utterance or a present sense
impression. See Point One. The victim’s statement was arguably an excited utterance because it
was made in response to an event (the victim hearing the defendant’s voice) that was evidently
“startling” to the victim, it occurred close in time to the event, and the victim was “under the
stress” of the excitement of the event when making the statement. Fed. r. evid. 803(2).

19
Evidence Analysis

Similarly, the victim’s statement was a present sense impression: it “describ[ed] . . . an event”
(hearing “the voice of the guy who robbed me”) and was made immediately after the event. Fed.
r. evid. 803(1).

Point Three (25%)

The police officer’s testimony that the defendant was a “known drug dealer who had been hang­
ing around in the area where the jewelry store is located for six months before the robbery,
constantly causing trouble” is inadmissible character evidence.

In relevant part, Federal Rule of Evidence 404(a)(1) provides that “[e]vidence of a person’s char­
acter or a character trait is not admissible to prove that on a particular occasion the person acted
in accordance with the character or trait.” Here, the only apparent purpose of the police officer’s
testimony is to impugn the defendant’s character and suggest that he is the sort of person who
would commit a robbery.

The testimony is not admissible under Federal Rule of Evidence 404(b)(2), which provides that
evidence of crimes, wrongs, or other acts “may be admissible for another [non-propensity] pur­
pose, such as proving motive, opportunity, intent, preparation, plan, knowledge, identity, absence
of mistake, or lack of accident.” It is unlikely that the police officer’s vague reference to the
defendant’s “constantly causing trouble” would be considered evidence of other crimes, wrongs,
or acts under Rule 404(b).

Even in the unlikely event that the judge concluded that a vague reference to causing trou­
ble was evidence of other crimes, wrongs, or acts, this evidence does not support any relevant
non-propensity issue. For example, some courts have allowed prosecutors to admit evidence
of a defendant’s drug habit to establish the motive for a crime (i.e., to prove that a defendant
needed to acquire money to purchase more drugs), see, e.g., United States v. Cody, 498 F.3d 582,
590–91 (6th Cir. 2007). Here, there is no indication that the defendant had a drug habit or that his
previous behavior provided a motive for this robbery.

20
AGENCY & PARTNERSHIP ANALYSIS
Agency and Partnership V.C.; VI.; VII.C., D.

ANALYSIS
Legal Problems:
(1) Is LLP liable to the bank on the loan undertaken by a partner acting beyond his actual
authority, but within the partnership’s ordinary course of business?
(2) Is the woman personally liable to the bank on the loan it made to LLP?
(3)(a) Did the man breach his fiduciary duties by entering into an unauthorized transaction
and appropriating partnership assets for his own use?
(3)(b) If the man breached any fiduciary duties, does the woman and/or LLP have a claim
against the man?

DISCUSSION
Summary

Although the man had no actual authority to enter into the loan with the bank, the man was act­
ing within the partnership’s ordinary course of business and thus had apparent authority to bind
LLP on the loan. Thus, LLP is liable on the loan.

The woman is not liable to the bank on the loan because a partner in a limited liability partner­
ship does not incur personal liability on partnership obligations solely by reason of being a
partner. The woman did not engage in any misconduct that would warrant an exception to this
rule of limited liability.

The man breached his fiduciary duties (duty of care and duty of loyalty) when he wrongfully
incurred LLP debt and misappropriated the $25,000 for his personal use. Either the woman or
the partnership can maintain an action against the man to recover damages for losses resulting
from the man’s breach. The woman can also bring an accounting action to have the partnership
recover damages for losses resulting from the man’s breach of his duty of loyalty.

[NOTE: Because this is a limited liability partnership, the question is analyzed under the Revised
Uniform Partnership Act, RUPA (1997, as revised 2013), not the prior act, Uniform Partnership
Act, UPA (1914). RUPA has been adopted in the following MEE jurisdictions: Alabama,
Arizona, Arkansas, Colorado, Connecticut, the District of Columbia, Hawaii, Idaho, Illinois,
Iowa, Kansas, Kentucky, Mississippi, Montana, Nebraska, Nevada, New Mexico, North Dakota,
Oregon, South Dakota, and West Virginia. The analysis would be substantially the same in states
that continue to use UPA (1914) and have adopted additional provisions for limited liability
partnerships.]

Point One (30%)

LLP is liable to the bank on the loan because the man, as a partner of LLP, had apparent
authority.

21
Agency & Partnership Analysis

Even if a partner lacks actual authority, a limited liability partnership can be bound by the acts of
a partner, “including the execution of an instrument in the partnership name,” if the partner was
“apparently carrying on in the ordinary course the partnership business or business of the kind
carried on by the partnership . . . .” RUPA § 301(1). From the bank’s perspective, the man acting
as a partner had apparent authority to incur the debt because borrowing $25,000 to pay for ordi­
nary maintenance expenses of a multi-million-dollar apartment complex was entirely consistent
with LLP’s ordinary business. This is evident from the facts that the partners anticipated the need
for such loans when they formed the LLP, the LLP had previously borrowed from the bank for
such maintenance expenses, and the bank had previously made similar loans to other apartment
complexes.

Apparent authority did not exist, however, if the bank had actual knowledge that the man lacked
authority. Under the RUPA (1997), a third party is bound by a limitation of authority only if that
party “knew or had notice that the partner lacked authority.” See RUPA § 301(1). Here, there
are no facts suggesting that the bank had notice or knowledge of the limitation on authority.
See RUPA § 103 (“knowledge” is generally limited to actual knowledge, and “notice of a fact”
arises from “all the facts known to the person at the time in question”). The bank asked the man
if he had authority and the man, in response, gave the bank a copy of the partnership agreement
containing no limits on his authority. The man did not give the bank a copy of the statement
of partnership authority evidencing the man’s lack of actual authority. Nor did the bank have
knowledge of this statement, which was never filed.

[NOTE: If an examinee concludes that borrowing $25,000 was not in the ordinary course of
business, then the examinee must conclude that there was no apparent authority either.]

Point Two (30%)

The woman is not personally liable to the bank on its claim under the loan agreement.

As a partner in LLP, the woman has limited liability for any partnership debts and thus is not
liable to the bank. RUPA provides that a partner in a limited liability partnership is not liable for
partnership obligations “solely by reason of being or acting as a partner.” RUPA § 306(c). The
loan obligation arises out of contract, and thus the woman would not be liable for it unless there
were some basis for asserting liability against her other than her being a partner.

Partners can become liable, however, for partnership obligations based on their own personal
misconduct. Here, there is no indication of fraudulent or inequitable conduct by the woman
that would justify liability for personal misconduct or piercing the entity veil. See Comments to
RUPA § 306(c), (d) (limited liability irrelevant to claim of partner’s own misconduct; doctrine of
“piercing the corporate veil” applies in limited liability partnerships); see also Red River Wings,
Inc. v. Hoot, Inc., 751 N.W.2d 206 (N.D. 2008) (“Principles for piercing a corporate veil apply to
limited liability partnerships”).

22
Agency & Partnership Analysis

[NOTE: Under some statutes, though not RUPA, partners in a limited liability part­
nership may also become liable for “the negligence, wrongful acts or misconduct of
any person under the partner’s direct supervision and control.” See ariz. rev. Stat. ann.
§ 29-215(C) (Supp. 1994); see also carter g. BiShop & daniel S. kleinBerger, liMited
liaBility coMpanieS: tax and BUSineSS law ¶ 15.02[3][e][iii] (2015) (collecting limited liability
partnership statutes that include supervisor liability). Under this type of statute, the bank may
argue that the woman failed to supervise the man when he entered into the loan without author­
ity. There is nothing, however, to suggest that the man warranted supervision or that the woman
was negligent for not supervising the man. In a partnership, each partner is deemed to be the
co-equal of the other partners, and no partner is under the control of the other partners. See
RUPA § 401(h) (“Each partner has equal rights in the management and conduct of the partner­
ship’s business”).]

Point Three(a) (20%)

By improperly obtaining the bank loan and then misappropriating the loan proceeds, the man
breached his fiduciary duty of loyalty and his duty of care.

Under RUPA, a partner owes to the partnership and the other partners the duties of loyalty and
care. RUPA § 409(a). Partners are liable for damages to the partnership and co-partners for
breach of these duties. Claims for breach of duties by partners in a limited liability partnership
are not subject to the rule of limited liability applicable to claims by outside parties. Here, the
man’s conduct breached his fiduciary duty of loyalty, as well as his broader duty of care.

The fiduciary duty of loyalty includes the obligation to refrain from appropriating partner­
ship assets for personal use. See RUPA § 404(b)(1) (partner’s duty of loyalty includes duty “to
account to the partnership . . . for . . . any property, profit, or . . . benefit derived by the partner in
the conduct . . . of the partnership business”). The man breached this duty by misappropriating
the proceeds of the loan from the bank made to the partnership.

The duty of care, which is remediable in damages, includes a duty not to engage in intentional
misconduct and knowing violations of law. See RUPA § 409(c) (partner’s duty of care includes
“refraining from . . . intentional misconduct, or a knowing violation of law”). The man breached
this duty by not mentioning or providing the loan officer with the statement of partnership
authority, which limited his ability to borrow more than $10,000 without the other partner’s
consent.

[NOTE: RUPA, as revised in 2013, treats a partner’s duty of loyalty as a fiduciary duty, the
breach of which gives rise to a full range of legal and equitable remedies. On the other hand, the
revision does not refer to a partner’s duty of care as a fiduciary duty, because “the duty of care
applies in many non-fiduciary situations.” See Comment to RUPA § 410(c) (describing label
change as “merely semantics, no change in the law is intended”). Thus, an examinee who refers
to the duty of care as a fiduciary duty in the context of this question should receive full credit.]

23
Agency & Partnership Analysis

Point Three(b) (20%)

The woman (or the partnership) can bring a direct action against the man for breaching his duties
of loyalty and care. The woman can also bring an accounting action seeking to have the man pay
damages to the partnership for his loyalty breach.

The duties of loyalty and care run to both “the partnership and the other partners.” See RUPA
§ 409(a); cf. RUPA § 404 (1997). Thus, the partnership can maintain an action against a partner
for violating his fiduciary duties to the partnership and thus causing harm to the partnership.
RUPA § 410(a). In addition, a partner can maintain an action against another partner, with or
without an accounting, to enforce the partner’s rights under the partnership act, including an
action for violations of duties. RUPA § 410(b).

Here, the woman can bring a direct action seeking to have the man make her whole for any
losses to her caused by his misconduct that breached his duties of loyalty and care. She can also
bring an accounting action to have the man account to the partnership for the money he took
from the partnership. Although the partnership could seek damages for these breaches as well,
in this two-person partnership it is unlikely that the man would agree to have the partnership
sue him.

[NOTE: Some examinees might conclude that the woman can bring a direct action only for the
man’s care breach, and the woman would have to pursue an accounting action with respect to the
loyalty breach. Although this seems to have been the approach of RUPA (1997), this approach
was abandoned in the 2013 revisions to ensure that partners can bring direct claims to protect
their interests. See Comments to RUPA §§ 409, 410(b). Finally, some examinees might point
out that derivative actions are not permitted on behalf of a partnership. See Comment to RUPA
§ 310(b) (noting that “statutory language does not contemplate derivative actions”); see also
Adams v. Land Services, Inc., 194 P.3d 429 (Colo. Ct. App. 2008) (holding that UPA (1914)
does not allow derivative actions). This analysis does not affect the conclusion that the woman
can bring a direct action against the man for breaching his duties of loyalty and care, given that
both of these duties run to both “the partnership and the other partners.” See RUPA § 409(a).]

24
CONSTITUTIONAL LAW ANALYSIS
Constitutional Law II.A.1.

ANALYSIS
Legal Problems:
(1) May a state enact a law that has the effect of favoring an in-state industry at the
expense of an out-of-state industry where there are environmental reasons to favor the
in-state industry?
(2) May a state deny an out-of-state utility a permit to construct a coal-burning power
plant because the plant, although it meets urgent out-of-state energy needs, does not
meet urgent energy needs of the permitting state?
(3) May a state favor in-state vendors when purchasing goods and services?

DISCUSSION
Summary

Even when Congress has not acted, the Commerce Clause of the United States Constitution
imposes by negative implication a limitation on state laws that discriminate against or unduly
burden interstate commerce.

Section 1 of the Act probably passes constitutional muster under the Commerce Clause. It is not
facially discriminatory because it applies equally to in-state and out-of-state utilities; and it does
not impede the import or export of electricity. The law’s incidental effect of favoring an in-state
industry (wind) at the expense of an out-of-state industry (natural gas production) probably does
not trigger strict scrutiny. Moreover, the burden on interstate commerce is not clearly excessive
in light of the in-state benefits.

Section 2 of the Act, as applied, probably does not pass constitutional muster. A state law or
administrative decision that explicitly discriminates against nonresidents violates the Commerce
Clause unless it is narrowly tailored to meet a legitimate, nonprotectionist purpose. The legisla­
tive ban on new coal-burning power plants, with its limited exception when State A has urgent
energy needs, is discriminatory because it blocks the export of coal-produced electricity to other
states. Even if reducing pollution from coal-burning power plants is a legitimate, nonprotection­
ist purpose, the permit denial likely fails strict scrutiny because there are other, nondiscrimina­
tory means to accomplish the purpose.

Section 3 of the Act should pass constitutional muster. The Commerce Clause does not limit
state action when the state acts as a “market participant.” Thus, State A may favor in-state ven­
dors when purchasing goods and services.

25
Constitutional Law Analysis

Point One (40%)

Section 1 of the Act, which requires utilities to use environmentally friendly energy sources, is
probably valid given that it is not facially discriminatory against out-of-state energy producers
and its discriminatory impact is not in the market being regulated (generation of electricity), but
instead affects another market (natural gas production). Further, the law appears to satisfy the
Pike balancing test, given that its burdens on interstate commerce are not clearly excessive in
light of the putative in-state benefits.

State laws that discriminate against out-of-state commerce in favor of in-state commerce—either
on their face or in practical effect—are subject to strict scrutiny and thus a nearly per se rule
of invalidity. Even if not discriminatory, state laws that affect interstate commerce can also be
invalidated if the burden on interstate commerce is clearly excessive in relation to the putative
in-state benefits. Pike v. Bruce Church, Inc., 397 U.S. 137 (1970).

Section 1 is not facially discriminatory. Utilities may meet the requirement that 50% of their
electricity come from environmentally friendly energy sources by acquiring electricity from out­
of-state wind or other environmentally friendly energy sources; natural gas does not qualify as an
environmentally friendly energy source regardless of where it is produced.

Section 1, however, may be discriminatory in practical effect because it favors an in-state


industry (wind) over an out-of-state industry (natural gas). See Hunt v. Washington State Apple
Advertising Comm’n, 432 U.S. 333 (1977) (invalidating state statute that imposed labeling
requirements on out-of-state apple producers, effectively advantaging in-state apple producers).
This discriminatory-impact argument, however, likely fails under Exxon Corp. v. Governor of
Maryland, 437 U.S. 117 (1978) (upholding ban on refiner-owned service stations by state in
which no refiners were located). In Exxon the Court read the Hunt discriminatory-impact test
to apply to a direct impact on out-of-state firms in the primary market (apples) regulated by
the state. Exxon, 437 U.S. at 126. In Exxon, the discriminatory impact was in a market (refin­
ing) different from the one regulated by the state (service stations), and so the state law was not
found to be discriminatory. Here, the discriminatory impact of Section 1 is felt in a market (natu­
ral gas production) different from the one being regulated (generation of electricity). See also
Minnesota v. Cloverleaf Creamery Co., 449 U.S. 456 (1981) (upholding state law requiring milk
to be sold in paper cartons, even though it favored in-state paper industry over out-of-state plas­
tics industry). Although evidence of protectionist motives (such as statements in the legislative
history) might be relevant to whether the law is discriminatory in practical effect, see Kassel v.
Consolidated Freightways Corp., 450 U.S. 662 (1981), the facts do not suggest any such motive.

Further, Section 1 does not appear to burden interstate commerce in ways that are clearly exces­
sive in relation to the putative in-state benefits. See Pike v. Bruce Church, Inc., supra. There
is no indication of an especially significant burden on interstate commerce. Cf. Bibb v. Navajo
Freight Lines, Inc., 359 U.S. 520 (1959) (invalidating state law obstructing flow of interstate
commerce); Edgar v. MITE Corp., 457 U.S. 624 (1982) (invalidating state law that imposed

26
Constitutional Law Analysis

extraterritorial regulations on out-of-state activities). Conversely, the findings of the legislature


indicate that the law’s goal is to promote environmentally friendly energy sources, which could
reduce air pollution and generate other significant local benefits (e.g., less use of water in elec­
tricity production).

Point Two (35%)

Section 2 of the Act, as applied by the Public Service Commission, is likely unconstitutional
because it discriminates against out-of-state consumers by preventing the export of electricity
from new coal-burning power plants. Although the environmental purposes of the law are legiti­
mate, the law is not narrowly tailored to meet them.

Section 2 of the Act, and the Public Service Commission’s denial of a permit for an out-of-state
utility’s coal-burning power plant, are discriminatory on their face. While a general ban on the
construction of coal-burning power plants would not be discriminatory because it would treat
resident and nonresident producers and consumers alike, the State A law creates an exception for
the urgent energy needs of state residents only. Thus, the law treats in-state electricity consumers
more favorably than out-of-state consumers and effectively bans the export of electricity from
new in-state coal-burning plants. See H.P. Hood & Sons, Inc. v. du Mond, 336 U.S. 525 (1949)
(invalidating denial of license for plant to process milk for export to another state).

The permit denial here discriminates against out-of-state consumers. If the application had been
for the sale of electricity to meet the urgent needs of consumers in State A, the application could
have been approved. Instead, it was denied because the State B utility only identified the urgent
needs of consumers in State B. The case is analogous to City of Philadelphia v. New Jersey,
437 U.S. 617 (1978), where the Court invalidated a New Jersey law prohibiting the disposal of
out-of-state waste in New Jersey landfills, effectively precluding the export of waste disposal
services and preferring in-state consumers. In City of Philadelphia, the Court made clear that it
does not matter whether the law has a legitimate environmental purpose; the state may not use
discriminatory means to accomplish it.

Insofar as the law is discriminatory, it is invalid unless it is narrowly tailored to meet a legiti­
mate, nonprotectionist purpose. See Maine v. Taylor, 477 U.S. 131 (1986) (upholding ban on
importation of live baitfish because of threat of parasites introduced into in-state waters). In par­
ticular, a law is not narrowly tailored if there are less discriminatory alternative means to accom­
plish the state’s purpose. See, e.g., Hughes v. Oklahoma, 441 U.S. 322 (1979). Thus, although
reducing air pollution from coal-burning plants (the apparent reason for Section 2) may be a
legitimate, nonprotectionist purpose, the law is not narrowly tailored. There are less discrimina­
tory alternatives that would better accomplish the state’s objectives, such as (1) strict environ­
mental regulation of all in-state coal-burning power plants, (2) an across-the-board ban on all
in-state coal-burning power plants (without any exception), and (3) an exception for such plants
for urgent energy needs that does not discriminate against out-of-state consumers.

27
Constitutional Law Analysis

Point Three (25%)

Section 3, even though it discriminates against out-of-state vendors by requiring the state to pre­
fer in-state vendors, is a valid exercise of the state’s role as a “market participant.”

The state may discriminate in favor of residents when buying or selling goods and services
because the state is acting as a “market participant” rather than as a regulator of an economic
activity. See Reeves v. Stake, 447 U.S. 429 (1980) (state-owned cement plant could confine sales
to state residents during cement shortage). Thus, State A may limit its purchases to vendors in
the state. See Hughes v. Alexandria Scrap Corp., 426 U.S. 794, 810 (1976) (state bounty for
scrap automobiles may favor in-state processors of junked vehicles). Thus, even though the out­
of-state vendor meets all of State A’s requirements for an “environmentally friendly” vendor,
State A is still entitled to favor in-state vendors over the out-of-state vendor.

28
DECEDENTS’ ESTATES ANALYSIS
Decedents’ Estates II.I.6.; IV.D.

ANALYSIS
Legal Problems:
(1) Did the patient’s son, her health-care agent designated under a durable power of attor­
ney, have the power to instruct the doctor to enter a DNR order?
(2) Could the patient’s son be found liable for the patient’s death in a wrongful death
action?
(3) Is the patient’s son barred from taking a share of the patient’s estate under the state
statute?

DISCUSSION
Summary

The attorney is incorrect on all counts.

An agent acting under a durable health-care power of attorney (POA) is authorized to make
health-care decisions on behalf of the principal whenever the principal lacks the capacity to make
a decision herself. A duly appointed agent may make decisions on behalf of the principal without
the concurrence of the principal’s relatives. A durable power is not dependent upon a particular
diagnosis and remains in effect until and unless rescinded by the principal. Here, because the
patient was unconscious and unable to make health-care decisions, the son, her duly appointed
agent, was authorized to direct the doctor to enter a DNR order, even though the patient signed
the power when faced with a different medical condition. The opposition of the patient’s other
children is irrelevant.

Typical health-care durable powers statutes provide that agents acting in “good faith” are
immune from civil suit (such as a wrongful death action). Here, because there is no evidence of
bad faith, the son is likely immune from liability for the patient’s death.

The statute which precludes distribution of a share of a decedent’s estate to an individual who
caused the decedent’s death should be deemed inapplicable to the son. On the facts, there is a
strong argument that the son did not intentionally kill his mother. Moreover, most durable pow­
ers statutes specify that a death resulting from the withholding of treatment at the agent’s direc­
tive is not to be treated as a homicide or an intentional taking of the patient’s life. The statute
also appears to be a form of slayer statute, which generally applies only to intentional and feloni­
ous killing.

Point One (40%)

An agent acting under a durable health-care POA is empowered to make health-care decisions
whenever the principal lacks capacity to make a health-care decision. Thus, the patient’s son
could authorize the doctor to enter the DNR order here.

29
Decedents’ Estates Analysis

All states have adopted statutes authorizing advance directives and durable health-care pow­
ers. An advance directive (sometimes called a living will) specifies the patient’s (non)treatment
preferences should he or she become incapacitated. A durable health-care POA empowers a
designated agent to make health-care decisions for the principal in the event of the principal’s
incapacity. See generally Bretton J. Horttor, A Survey of Living Will and Advanced Health Care
Directives, 74 N.D. L. rev. 233 (1998) (surveying the relevant laws of all states).

Unless a durable POA specifies otherwise, a designated agent is empowered to make health-care
decisions for the principal whenever the principal lacks capacity; the power is not limited to a
particular illness or for a particular time period. See UniF. health-care deciSionS act §§ 2(c),
4; see also § 1(5) (discussed in Point Two below). Here, there is no language in the power condi­
tioning the agent’s authority to act for the principal except the patient’s incapacity. Thus, the fact
that the patient was motivated to execute the power because of a cancer diagnosis does not limit
her designated agent’s power to cancer-related decisions.

In the absence of specific instructions, a designated agent shall make decisions for the principal
“in accordance with the agent’s determination of the principal’s best interest . . . [considering]
the principal’s personal values to the extent known to the agent.” UniF. health-care deciSionS
act § 2(e). Here, the facts specify that the patient’s son was confident that his mother would not
want to be kept on life support if permanently unconscious. Given that a cardiac arrest would
substantially increase the already high risk of permanent unconsciousness, the son’s instruc­
tion not to resuscitate appears to be consistent with the patient’s best interest. This conclusion
is arguably supported by the fact that the doctor does not seem to have disagreed with entry of a
DNR order. Indeed, if the doctor did not believe that entry of a DNR order was consistent with
the patient’s interests and the applicable standard of care, he could—and probably should—have
declined to issue a DNR order. See id. § 7(f ).

Had the patient not executed a durable health-care power, a majority of the siblings would have
had to agree upon health-care decisions for their mother. See id. § 5(e). However, when, as here,
an agent has been designated by the patient, the agent may act on behalf of the patient without
consulting family members who are not designated agents of the patient. See id. § 5(a).

[NOTE: In some cases of family conflict about treatment for an incapacitated prin­
cipal, courts have required a surrogate decision maker to provide clear and convincing evi­
dence of the patient’s wishes as a precondition to withdrawing life-sustaining treatment. See
In re Martin, 450 Mich. 204, 538 N.W.2d 399 (1995); In re Wendland, 26 Cal. 4th 519 (2001).
However, in none of the reported cases did the surrogate hold a durable power, and at least one
state high court has noted that the clear and convincing evidence requirement is not applicable to
an agent with a durable power. Wendland, supra.]

Point Two (30%)

An agent who acts in good faith is not liable for damages in a wrongful death action. Here,
because there is no evidence that the son did not act in good faith, he would not be liable for
damages in a wrongful death action.

30
Decedents’ Estates Analysis

State laws governing durable health-care powers of attorney typically insulate an agent who
has acted in good faith from civil and criminal liability. See, e.g., UniF. health-care deciSionS
act § 9(b) (“An individual acting as an agent . . . is not subject to civil or criminal liability . . .
for health-care decisions made in good faith.”). A health-care decision includes directions “to
provide, withhold, or withdraw . . . all other forms of health care.” Id. § 1(6). Health care means
“any care, treatment, service or procedure to maintain, diagnose, or otherwise affect an individu­
al’s physical . . . condition.” Id. § 1(5).

Here, the son appears to have acted in good faith. The facts specify that the son was confident
that the patient, his mother, would not want to be maintained in a permanently unconscious con­
dition, there was more than a 50 percent chance of permanent unconsciousness, the patient was
of advanced age, both cardiac arrest and stroke presented the risk of additional brain damage,
and the doctor’s acquiescence suggests that a DNR order was consistent with the standard of
care applicable to the patient. There are no facts suggesting that the son acted in bad faith. The
fact that the patient named her son as agent may also indicate that she trusted his judgment and
understanding of her values.

Point Three (30%)

The son is not barred from claiming a share of the patient’s estate. The statute bars claimants
who “intentionally” cause the decedent’s death, which the son did not do. Moreover, the son is
not the type of claimant the statute is intended to bar.

The state statute provides: “No person shall share in the estate of a decedent when he or she
intentionally caused the decedent’s death.” The statute appears to be a so-called “slayer statute,”
which typically bars those who have feloniously and intentionally killed a decedent from taking
a portion of the decedent’s estate. See, e.g., UniF. proBate code § 2-803(b) (person who “feloni­
ously and intentionally kills the decedent forfeits all benefits . . . ”).

Here, the statute applies to slayers who “intentionally cause[] the decedent’s death,” creating
an argument that it applies to the patient’s son. Thus, statutes such as UniForM health-care
deciSionS act § 13(b), providing that “[d]eath resulting from the withholding . . . of health care
in accordance with this [Act] does not for any purpose constitute suicide or homicide . . . ,”
do not resolve the question whether an actor “caused” a death. However, there is a strong argu­
ment that the patient’s son did not “intentionally cause” his mother’s death as that phrase is
used in the statute. Moreover, factually the son did not cause his mother’s death. The son speci­
fied only that medical personnel should not resuscitate the patient in given circumstances, and
the doctor agreed and entered a DNR order on the patient’s chart. The immediate cause of the
patient’s death was cardiac arrest, and it is not even clear that a resuscitation attempt would
have been successful. Moreover, medical ethicists do not typically regard withholding of treat­
ment as the cause of death, and this perspective has found judicial support. See, e.g., Woods v.
Commonwealth, 142 S.W.3d 24, 45 (Ky. 2004) (“In letting die, the cause of death is seen as the
underlying disease process or trauma”); accord, Baxter v. State, 224 P.3d 1211 (Mont. 2009).

31
Decedents’ Estates Analysis

Applying the state slayer statute to the son would also be inconsistent with the typical immunity
provision in power of attorney statutes. For example, Uniform Health-Care Decisions Act § 9
immunizes agents acting in good faith from civil liability (see Point Two), and disinheritance is
akin to civil liability, as both result in the loss of assets to which the penalized individual is oth­
erwise entitled. Applying the statute to the son would also undermine an important purpose of
durable power of attorney statutes, ensuring that individuals can choose who makes decisions on
their behalf in the event of incapacity. Individuals typically choose as agents close relatives who
are potential heirs or distributees; many of these individuals would be reluctant to serve if they
stood to lose a share of the estate based on making a good-faith decision which led to the princi­
pal’s death.

Thus, the son should not be barred from taking a share of his mother’s estate.

[NOTE: An examinee who concludes that the patient’s son caused, or contributed to, his moth­
er’s death but finds that the slayer statute does not apply should receive credit.]

32
FAMILY LAW ANALYSIS
Family Law III.D.

ANALYSIS
Legal Problems:
(1) Is the premarital agreement enforceable under the Uniform Premarital Agreement Act?
(2)(a) What assets owned by the man and the woman are subject to division at divorce?
(2)(b) Under an equitable distribution rule, what factors will a court consider in distributing
the couple’s divisible assets?

DISCUSSION
Summary

Under the UPAA, adopted in this state, a premarital agreement will be enforced unless the party
against whom enforcement is sought (1) did not execute the agreement voluntarily or (2) both
(a) did not receive or waive fair and reasonable disclosure of assets of the other party and (b)
can show that the agreement was unconscionable when executed. Here, the agreement is likely
enforceable because the woman fully disclosed her assets and the facts do not support a finding
that the man entered into the agreement involuntarily.

At divorce, most states permit the division of only marital or community property (i.e., prop­
erty acquired during the marriage except by gift, descent, or devise). Marital property typically
continues to accrue until the date a divorce is final, although some states use an earlier date,
typically the date of separation or divorce filing. An increase in the value of a premarital asset
resulting from significant spousal effort during marriage is divisible; an increase in the value of a
premarital asset resulting solely from market appreciation remains separate. Thus, the woman’s
lottery winnings are probably divisible because the woman purchased the winning ticket before a
divorce action was filed. The fact that the woman may receive the lottery winnings after divorce
does not alter this result. The woman’s condominium is not divisible marital property because it
was acquired before marriage and its increased value is due to market forces. Some portion of
the woman’s brokerage account is divisible, as the woman made contributions with marital earn­
ings. At least some of the man’s future royalties are divisible because the man expended signifi­
cant effort during the marriage on the novel from which those royalties will derive. Again, the
fact that the man will receive the royalties after divorce does not alter their divisibility at divorce.

When equitably dividing marital property, courts consider need, contribution (both monetary and
nonmonetary), and marital duration. Here, the marriage was short, the man is the needier spouse,
and the woman has made larger financial contributions to the marriage. It is difficult to ascertain
who made more nonmonetary contributions to the marriage.

Point One (35%)

Under the UPAA, a premarital agreement entered into after full disclosure of assets and obliga­
tions is binding unless it was entered into involuntarily. Here, the woman fully disclosed her
33
Family Law Analysis

assets, and the facts do not support a finding that the man signed the agreement involuntarily.
Thus, the agreement is likely enforceable.

Although courts were once hostile to premarital agreements, today all states permit spouses to
contract premaritally with respect to rights and obligations in property whether acquired before
or during the marriage. In all states, the enforceability of such an agreement turns on three fac­
tors: voluntariness, fairness, and disclosure. How courts apply these factors varies significantly
from one state to the next. In many states, an agreement is unenforceable if the party against
whom enforcement is sought succeeds in showing either involuntariness, unfairness, or lack of
adequate disclosure.

However, under the UPAA, which has been adopted in 25 states and the state in which this
question is set, the party against whom enforcement is sought must prove (1) involuntariness or
(2) both that “the agreement was unconscionable when it was executed” and that he or she did
not receive or waive a “fair and reasonable” disclosure and “did not have or reasonably could not
have had . . . an adequate knowledge” of the other’s assets and obligations. UPAA § 6(a). Thus, a
court may not refuse to enforce a premarital agreement based on substantive unfairness unless it
also finds lack of adequate disclosure or knowledge.

Here, the man cannot show inadequate disclosure because the woman gave him an accurate asset
list with the agreement. Under the UPAA, the man thus must show that the execution of the
agreement was involuntary in order to avoid its enforcement.

In considering whether a premarital agreement was voluntarily executed, courts look to whether
there was fraud, duress, or coercion. They agree that one party’s insistence on signing the
agreement as a condition of the marriage does not, of itself, render the agreement involuntary,
but there is no consensus on what additional facts are sufficient to establish involuntariness.
See principleS oF the law oF FaMily diSSolUtion: analySiS & recoMMendationS § 7.04
cmt. c (2002). Many of the reported cases, like this one, involve a claim of involuntariness based
on presentation of an agreement very close to the wedding. In analyzing whether an agreement
signed under these circumstances is voluntary, courts have looked at a wide range of factors,
including the difficulty of conferring with independent counsel, other reasons for proceeding
with the marriage (for example, a preexisting pregnancy), and financial losses and embarrass­
ment arising from cancellation of the wedding. The older cases tend to be more sympathetic to a
spouse who receives a last-minute demand; “[i]n recent years, more courts have accepted [last­
minute] agreements.” See id. § 7.04 cmt. d, reporter’s notes.

Here, the woman presented the agreement to the man only a week before the wedding. But the
man had time to confer with independent counsel, and he did so. The man ultimately rejected
the advice of the lawyer he consulted. The man and the woman were married at City Hall, and
there are no facts suggesting any hardship that the man would have suffered from canceling or
postponing the ceremony. On these facts, it is unlikely that the man can establish involuntariness.
Under the UPAA, the premarital agreement thus is likely enforceable.

34
Family Law Analysis

Point Two(a) (50%)

Assets acquired before marriage or by inheritance are separate, not marital, property. In most
jurisdictions, neither separate property nor appreciation in its value is subject to division at
divorce. However, significant spousal effort during the marriage that adds value to a separate
asset does create divisible marital property. Thus, the woman’s condominium is separate and
nondivisible; the woman’s lottery winnings are marital and divisible; and some portion of
the woman’s brokerage account and at least some of the man’s royalties are also marital and
divisible.

In all states (whether community property or common law jurisdictions), a divorce court may
divide marital (community) property without regard to title. In this and the vast majority of
jurisdictions, a divorce court may not divide separate property. In a minority of jurisdictions—
so-called “hotchpot” jurisdictions—the court may divide all assets, whenever or however
acquired; a few states permit the division of separate property in special circumstances, such
as hardship. See J. thoMaS oldhaM, divorce, Separation and the diStriBUtion oF property
§ 13.02[i] n.66 (2006) (listing state regimes).

An asset is marital if it was acquired during the marriage by any means other than gift, descent,
or devise. In the majority of states, marital property continues to accrue until a final divorce
decree is entered. See harry d. kraUSe, linda d. elrod, MarSha garriSon, and j. thoMaS
oldhaM, FaMily law caSeS, coMMentS, and QUeStionS 756 (6th ed. 2007). In some states, how­
ever, marital property ceases to accrue after the date of permanent separation (see, e.g., N.C.
gen. Stat. § 50-20) or the date of divorce filing (see, e.g., N.Y. doM. rel. L. § 236B). However,
expectancies created during the marriage are still marital even if payment will not be received
until after the marriage ends. Even contingent expectancies such as unvested pension rights are
subject to division at divorce if they were acquired through spousal effort during the marriage.
See kraUSe et al., supra, note at 773–76.

An asset that is initially separate property may be partially transformed into divisible marital
property if marital funds or significant effort by the owner-spouse enhance its value or build
equity. In the vast majority of states, the appreciation of separate assets during the marriage does
not create marital property. See J. thoMaS oldhaM, supra, § 5.03(4). But see colo. rev. Stat.
§ 14-10-113.

Here, unless the woman and the man live in a state in which marital property ceases to accrue
after separation, the woman’s lottery award is marital property because the woman purchased
the winning ticket before the man filed for divorce. It does not matter that the woman may not
receive any winnings before the divorce filing.

The entire value of the woman’s condominium is separate property because she purchased the
condominium outright before marriage, and the facts indicate that its increased value is due
entirely to market forces, not spousal efforts or marital funds.

35
Family Law Analysis

Some portion of the woman’s brokerage account is divisible marital property because the woman
contributed her employment bonuses during the marriage to that account; only the portion attrib­
utable to investments the woman made before marriage remains separate.

Some, if not all, of the royalties the man will receive in the future are divisible marital prop­
erty despite the fact that the man began the novel before marriage. Significant spousal effort
during marriage that enhances the value of a separate asset creates marital property, and the
man worked at paid employment only part-time throughout the marriage in order to work on the
novel. The facts do not specify how much the man worked on the novel during the marriage. If
the man completed a significant portion of the novel during the marriage, the value of the novel
would be apportioned into separate and marital components. See kraUSe et al., supra, at 763;
Cockrill v. Cockrill, 601 P.2d 1334 (Ariz. 1979) (describing apportionment principles). It is
irrelevant that the royalties will not be received until after the marriage ends. See kraUSe et al.,
supra, at 756.

[NOTE: The above analysis applies in both common law and community property jurisdictions.
Although the rules governing asset management and distribution at death vary depending on
whether the jurisdiction is a common law or community property state, today, all states disregard
title in defining the pool of assets available for divorce. Indeed, the marital property rules appli­
cable in common law states are sometimes referred to as “deferred” community property.]

Point Two(b) (15%)

In dividing marital assets, divorce courts look at contribution (monetary and nonmonetary), need,
and marital duration. Here, the marriage was short, the man is the needier spouse, and the woman
made greater financial contributions to the marriage, contributions which may have enabled the
man to complete his novel.

In an equitable distribution state, courts have broad authority to divide property fairly. Typically,
either by statute or by case law, courts are given a list of relevant, non-exclusive factors to con­
sider. Relevant factors usually include the duration of the marriage; indicators of each spouse’s
future needs (including age, health, resources, and occupational opportunities); and contributions
of the parties to the marriage and the acquisition of assets. See oldhaM, supra, § 13.02[1].

Here, the woman appears to have made much larger financial contributions to the marriage than
the man, and she appears to be better off now than the man because of her occupation as an
investment banker and her lottery winnings. The couple has no children, so the man has made
no significant nonfinancial contributions as a parent; there are also no facts to suggest that he has
made significant nonfinancial contributions as a homemaker. Instead, the woman’s support freed
the man to work on his novel.

The broad discretion given courts to distribute property equitably often makes it impossible to
predict with certainty how a court will divide marital property. See ira Mark ellMan et al.,
FaMily law: caSeS, text, proBleMS 323 (5th ed. 2010).

36
Family Law Analysis

[NOTE: An examinee may appropriately note that the uncertain value of the man’s expected roy­
alties may play a role in determining how a property award should be fashioned. With a pension
of uncertain value to be paid in the future, courts often order payment of a specified percentage
of pension payments when those payments are actually made instead of requiring the title-
holding spouse to compensate the other spouse at the time of divorce.]

37
NOTES

______________________________________________________________________________________________________________________________________

______________________________________________________________________________________________________________________________________

______________________________________________________________________________________________________________________________________

______________________________________________________________________________________________________________________________________

______________________________________________________________________________________________________________________________________

______________________________________________________________________________________________________________________________________

______________________________________________________________________________________________________________________________________

______________________________________________________________________________________________________________________________________

______________________________________________________________________________________________________________________________________

______________________________________________________________________________________________________________________________________

______________________________________________________________________________________________________________________________________

______________________________________________________________________________________________________________________________________

______________________________________________________________________________________________________________________________________

______________________________________________________________________________________________________________________________________

______________________________________________________________________________________________________________________________________

______________________________________________________________________________________________________________________________________

______________________________________________________________________________________________________________________________________

______________________________________________________________________________________________________________________________________

______________________________________________________________________________________________________________________________________

______________________________________________________________________________________________________________________________________

______________________________________________________________________________________________________________________________________

______________________________________________________________________________________________________________________________________

______________________________________________________________________________________________________________________________________

______________________________________________________________________________________________________________________________________

______________________________________________________________________________________________________________________________________

______________________________________________________________________________________________________________________________________

______________________________________________________________________________________________________________________________________

______________________________________________________________________________________________________________________________________

______________________________________________________________________________________________________________________________________
NOTES

______________________________________________________________________________________________________________________________________

______________________________________________________________________________________________________________________________________

______________________________________________________________________________________________________________________________________

______________________________________________________________________________________________________________________________________

______________________________________________________________________________________________________________________________________

______________________________________________________________________________________________________________________________________

______________________________________________________________________________________________________________________________________

______________________________________________________________________________________________________________________________________

______________________________________________________________________________________________________________________________________

______________________________________________________________________________________________________________________________________

______________________________________________________________________________________________________________________________________

______________________________________________________________________________________________________________________________________

______________________________________________________________________________________________________________________________________

______________________________________________________________________________________________________________________________________

______________________________________________________________________________________________________________________________________

______________________________________________________________________________________________________________________________________

______________________________________________________________________________________________________________________________________

______________________________________________________________________________________________________________________________________

______________________________________________________________________________________________________________________________________

______________________________________________________________________________________________________________________________________

______________________________________________________________________________________________________________________________________

______________________________________________________________________________________________________________________________________

______________________________________________________________________________________________________________________________________

______________________________________________________________________________________________________________________________________

______________________________________________________________________________________________________________________________________

______________________________________________________________________________________________________________________________________

______________________________________________________________________________________________________________________________________

______________________________________________________________________________________________________________________________________

______________________________________________________________________________________________________________________________________
NOTES

______________________________________________________________________________________________________________________________________

______________________________________________________________________________________________________________________________________

______________________________________________________________________________________________________________________________________

______________________________________________________________________________________________________________________________________

______________________________________________________________________________________________________________________________________

______________________________________________________________________________________________________________________________________

______________________________________________________________________________________________________________________________________

______________________________________________________________________________________________________________________________________

______________________________________________________________________________________________________________________________________

______________________________________________________________________________________________________________________________________

______________________________________________________________________________________________________________________________________

______________________________________________________________________________________________________________________________________

______________________________________________________________________________________________________________________________________

______________________________________________________________________________________________________________________________________

______________________________________________________________________________________________________________________________________

______________________________________________________________________________________________________________________________________

______________________________________________________________________________________________________________________________________

______________________________________________________________________________________________________________________________________

______________________________________________________________________________________________________________________________________

______________________________________________________________________________________________________________________________________

______________________________________________________________________________________________________________________________________

______________________________________________________________________________________________________________________________________

______________________________________________________________________________________________________________________________________

______________________________________________________________________________________________________________________________________

______________________________________________________________________________________________________________________________________

______________________________________________________________________________________________________________________________________

______________________________________________________________________________________________________________________________________

______________________________________________________________________________________________________________________________________

______________________________________________________________________________________________________________________________________
National Conference of Bar Examiners
302 South Bedford Street | Madison, WI 53703-3622

® Phone: 608-280-8550 | Fax: 608-280-8552 | TDD: 608-661-1275


www.ncbex.org e-mail: [email protected]

You might also like